Function Minimum Value
Let $ x, $ $ y, $ $ z $ be positive real numbers. Find the minimum value of
\[\frac{(1 + 5z)(4z + 3x)(5x + 6y)(y + 18)}{xyz}.\]
- 1
- 2
- 3
- +
- 4
- 5
- 6
- -
- 7
- 8
- 9
- $\frac{a}{b}$
- .
- 0
- =
- %
- $a^n$
- $a^{\circ}$
- $a_n$
- $\sqrt{}$
- $\sqrt[n]{}$
- $\pi$
- $\ln{}$
- $\log$
- $\theta$
- $\sin{}$
- $\cos{}$
- $\tan{}$
- $($
- $)$
- $[$
- $]$
- $\cap$
- $\cup$
- $,$
- $\infty$
Solution
We write
\begin{align*}
\frac{(1 + 5z)(4z + 3x)(5x + 6y)(y + 18)}{xyz} &= \frac{4}{5} \cdot \frac{(1 + 5z)(5z + \frac{15}{4} x)(5x + 6y)(y + 18)}{xyz} \\
&= \frac{4}{5} \cdot \frac{4}{3} \cdot \frac{(1 + 5z)(5z + \frac{15}{4} x)(\frac{15}{4} z + \frac{9}{2} y)(y + 18)}{xyz} \\
&= \frac{4}{5} \cdot \frac{4}{3} \cdot \frac{2}{9} \cdot \frac{(1 + 5z)(5z + \frac{15}{4} x)(\frac{15}{4} x + \frac{9}{2} y)(\frac{9}{2} y + 81)}{xyz} \\
&= \frac{32}{135} \cdot \frac{(1 + 5z)(5z + \frac{15}{4} x)(\frac{15}{4} x + \frac{9}{2} y)(\frac{9}{2} y + 81)}{xyz}.\end{align*}Let $ a = 5z, $ $ b = \frac{15}{4} x, $ and $ c = \frac{9}{2} y, $ so $ z = \frac{1}{5} a, $ $ x = \frac{4}{15} b, $ and $ y = \frac{2}{9} c $. Then
\begin{align*}
\frac{32}{135} \cdot \frac{(1 + 5z)(5z + \frac{15}{4} x)(\frac{15}{4} x + \frac{9}{2} y)(\frac{9}{2} y + 81)}{xyz} &= \frac{32}{135} \cdot \frac{(1 + a)(a + b)(b + c)(c + 81)}{\frac{4}{15} b \cdot \frac{2}{9} c \cdot \frac{1}{5} a} \\
&= 20 \cdot \frac{(1 + a)(a + b)(b + c)(c + 81)}{abc} \\
&= 20 \cdot (1 + a) \left( 1 + \frac{b}{a} \right) \left( 1 + \frac{c}{b} \right) \left( 1 + \frac{81}{c} \right).\end{align*}By AM-GM,
\begin{align*}
1 + a &= 1 + \frac{a}{3} + \frac{a}{3} + \frac{a}{3} \ge 4 \sqrt[4]{\left( \frac{a}{3} \right)^3}, \\
1 + \frac{b}{a} &= 1 + \frac{b}{3a} + \frac{b}{3a} + \frac{b}{3a} \ge 4 \sqrt[4]{\left( \frac{b}{3a} \right)^3}, \\
1 + \frac{c}{b} &= 1 + \frac{c}{3b} + \frac{c}{3b} + \frac{c}{3b} \ge 4 \sqrt[4]{\left( \frac{c}{3b} \right)^3}, \\
1 + \frac{81}{c} &= 1 + \frac{27}{c} + \frac{27}{c} + \frac{27}{c} \ge 4 \sqrt[4]{\left( \frac{27}{c} \right)^3},
\end{align*}so
\begin{align*}
20 \cdot (1 + a) \left( 1 + \frac{b}{a} \right) \left( 1 + \frac{c}{b} \right) \left( 1 + \frac{81}{c} \right) &\ge 20 \cdot 256 \sqrt[4]{\left( \frac{a}{3} \right)^3 \cdot \left( \frac{b}{3a} \right)^3 \cdot \left( \frac{c}{3b} \right)^3 \cdot \left( \frac{27}{c} \right)^3} \\
&= 5120.\end{align*}Equality occurs when
\[1 = \frac{a}{3} = \frac{b}{3a} = \frac{c}{3b} = \frac{27}{c},\]or $ a = 3, $ $ b = 9, $ and $ c = 27, $ which means $ x = \frac{12}{5}, $ $ y = 6, $ and $ z = \frac{3}{5} $. Therefore, the minimum value is $ \boxed{5120} $.